Administración     

Olimpiadas de Matemáticas
Página de preparación y problemas

Selector
La base de datos contiene 1154 problemas y 775 soluciones.
OME Local
OME Nacional
OIM
OME Andalucía
Retos UJA
Problema 290
En el plano se tienen un punto $P$ y un polígono $\Omega$ (no necesariamente convexo). Sea $\ell$ el perímetro de $\Omega$, $d$ la suma de las distancias desde $P$ a los vértices de $\Omega$ y $h$ la suma de las distancias de $P$ a las rectas que contienen a los lados de $\Omega$. Demostrar que $d^2-h^2\geq\frac{1}{4}\ell^2$.
Sin pistas
Sin soluciones
info
Si crees que el enunciado contiene un error o imprecisión o bien crees que la información sobre la procedencia del problema es incorrecta, puedes notificarlo usando los siguientes botones:
Informar de error en enunciado Informar de procedencia del problema
Problema 289
Sea $ABC$ un triángulo acutángulo y supongamos que las rectas tangentes en $A$ y $B$ a su circunferencia circunscrita cortan en los puntos $T$ y $U$, respectivamente, a la tangente a esta circunferencia en el punto $C$. Sean $P$ el punto de corte de las rectas $AT$ y $BC$ y $Q$ el punto medio de $AP$ , $R$ el punto de corte de las rectas $BU$ y $CA$ y $S$ el punto medio de $BR$. Demostrar que $\angle ABQ=\angle BAS$ y determinar, en términos de las razones de las longitudes de los lados, los triángulos para los que dicho ángulo es máximo.
Sin pistas
Sin soluciones
info
Si crees que el enunciado contiene un error o imprecisión o bien crees que la información sobre la procedencia del problema es incorrecta, puedes notificarlo usando los siguientes botones:
Informar de error en enunciado Informar de procedencia del problema
Problema 288
Dados números reales $x,y,z>0$ tales que $xyz=1$, demostrar que \[\frac{x^3}{(1+y)(1+z)}+\frac{y^3}{(1+z)(1+x)}+\frac{z^3}{(1+x)(1+y)}\geq\frac{3}{4}.\]
pistasolución 1info
Pista. La desigualdad de Chebyshev puede ser útil para transformar la desigualdad inicial.
Solución. Como la desigualdad es simétrica, podemos suponer que $x\geq y\geq z$ sin perder generalidad, de donde deducimos fácilmente que \[x^3\leq y^3\leq z^3,\qquad \frac{1}{(1+y)(1+z)}\leq \frac{1}{(1+x)(1+z)}\leq \frac{1}{(1+x)(1+y)}.\] De esta forma, podemos aplicar la desigualdad de Chebyshev, que nos asegura que \begin{eqnarray} \frac{x^3}{(1+y)(1+z)}+\frac{y^3}{(1+x)(1+z)}+\frac{z^3}{(1+x)(1+y)}&\geq&\frac{(x^3+y^3+z^3)\left(\frac{1}{(1+y)(1+z)}+\frac{1}{(1+x)(1+z)}+\frac{1}{(1+x)(1+y)}\right)}{3}\\ &=&\frac{(x^3+y^3+z^3)(3+x+y+z)}{3(1+x)(1+y)(1+z)} \end{eqnarray} Las desigualdades entre las medias aritmética y cúbica y entre las medias aritmética y geométrica nos permiten acotar \begin{eqnarray} x^3+y^3+z^3&\geq&\frac{1}{9}(x+y+z)^3,\\ 3(1+x)(1+y)(1+z)&\leq&\frac{1}{9}(3+x+y+z)^3 \end{eqnarray} Con estas dos desigualdades podemos transformar la desigualdad previa en \[\frac{x^3}{(1+y)(1+z)}+\frac{y^3}{(1+x)(1+z)}+\frac{z^3}{(1+x)(1+y)}\geq\frac{(x+y+z)^3}{(3+x+y+z)^2}.\] Ahora bien, la función $f(a)=\frac{a^3}{(3+a)^2}$ es estrictamente creciente para $a\geq 0$. Como $x+y+z\geq3\sqrt[3]{xyz}=3$, deducimos que $f(x+y+z)\geq f(3)=\frac{3}{4}$, de donde se deduce la desigualdad del enunciado.

Nota. Si la igualdad se alcanza, entonces del último razonamiento llegamos a que $x+y+z=3$, luego la igualdad en la desigualdad entre las medias nos asegura que $x=y=z=1$. Se comprueba que estos valores dan la igualdad y, por tanto, son los únicos.

Si crees que el enunciado contiene un error o imprecisión o bien crees que la información sobre la procedencia del problema es incorrecta, puedes notificarlo usando los siguientes botones:
Informar de error en enunciado Informar de procedencia del problema
Problema 287
Dado un punto $D$ interior al lado $BC$ del triángulo $ABC$, sea $X$ el otro punto de corte de la circunferencia circunscrita a $ABC$ con la recta $AD$ y sean $P$ y $Q$ los pies de las perpendiculares desde $X$ a las rectas $AB$ y $AC$, respectivamente. Demostrar que la recta $PQ$ es tangente a la circunferencia que tiene por diámetro $XD$ si, y sólo si, $AB=AC$.
Sin pistas
Sin soluciones
info
Si crees que el enunciado contiene un error o imprecisión o bien crees que la información sobre la procedencia del problema es incorrecta, puedes notificarlo usando los siguientes botones:
Informar de error en enunciado Informar de procedencia del problema
Problema 286
Calcula todos los números enteros $a$, $b$ y $c$ tales que $a^2=2b^2+3c^2$.
pistasolución 1info
Pista. Trabaja módulo $3$ y demuestra que los tres números deben múltiplos de $3$. Simplifica los factores $9$ comunes y vuelve a empezar.
Solución. Cualquier cuadrado es congruente con $0$ o $1$ módulo $3$. Por lo tanto, la ecuación solo es factible módulo $3$ si $a\equiv 0\ (\text{mod }3)$ y $b\equiv 0\ (\text{mod }3)$. Esto nos dice que podemos escribir $a=3x$ y $b=3y$ para ciertos $x,y\in\mathbb{Z}$. Sustituyendo, llegamos a que $9x^2=18y^2+3c^2$, luego $c^2=3x^2-6y^2$ debe ser también múltiplo de $3$, es decir, existe $z\in\mathbb{Z}$ tal que $c=3z$ y llegamos a otra solución de la misma ecuación: $x^2=2y^2+3z^2$.

Vamos a ver que esto implica que $a=b=c=0$. En efecto, si alguno de los tres números $a,b,c\in\mathbb{Z}$ es no nulo, entonces podríamos haber comenzado suponiendo que la solución $(a,b,c)$ es tal que la suma $a^2+b^2+c^2\gt 0$ es lo más pequeña posible (de entre todas las soluciones no nulas habrá una que cumpla esto), pero entonces $(x,y,z)$ es una solución con \[x^2+y^2+z^2=\frac{1}{9}(a^2+b^2+c^2)\lt a^2+b^2+c^2,\] en contradicción con el hecho de que $a^2+b^2+c^2$ es mínimo. Esto es lo que se llama técnica del descenso infinito o principio de minimalidad.

Si crees que el enunciado contiene un error o imprecisión o bien crees que la información sobre la procedencia del problema es incorrecta, puedes notificarlo usando los siguientes botones:
Informar de error en enunciado Informar de procedencia del problema
José Miguel Manzano © 2010-2024. Esta página ha sido creada mediante software libre